UMVUE of $e^\theta$ when $X_1, X_2, \dots, X_n$ are i.i.d. Uniform[$0, \theta$]

probability distributionsstatistical-inferencestatistics

$X_1, X_2, \dots, X_n$ are i.i.d. Uniform[$0, \theta$].

I was able to get that the order statistic $Y_n$ is sufficient and complete. How do I get the UMVUE of $e^\theta$?

I was thinking of doing something like $e^{Y_n}$ but then I got stuck.

Best Answer

Indeed, for a sample $(X_1,X_2,\ldots,X_n)$ from the $U(0,\theta)$ distribution, a complete sufficient statistic for the family is $$T(X_1,X_2,\ldots,X_n)=X_{(n)}$$

, the density of $T$ being $$g_{\theta}(t)=\frac{n}{\theta^n}t^{n-1}\mathbf1_{0<t<\theta}$$

So by Lehmann-Scheffe, an unbiased estimator of $e^\theta$ based on $T$ will be the UMVUE of $e^\theta$.

Let $h(T)$ be the required unbiased estimator.

Then, for all $\theta>0$,

\begin{align} \qquad\quad\frac{n}{\theta^n}\int_0^{\theta}h(t)t^{n-1}\,dt&=e^\theta \\\implies \int_0^{\theta}h(t)t^{n-1}\,dt &= \frac{\theta^ne^\theta}{n} \end{align}

Differentiating both sides wrt $\theta$,

\begin{align} h(\theta)\theta^{n-1}&=\frac{(n+\theta)\theta^{n-1}e^\theta}{n} \\\implies h(\theta)&=\frac{(n+\theta)e^{\theta}}{n} \end{align}

Hence your UMVUE must be $$h(T)=\frac{(n+T)e^T}{n}$$

This method to find the UMVUE works assuming that the UMVUE already exists.